Finding transformation T such that T(D*)=D

Click For Summary
To find the transformation T that maps the parallelogram D* with vertices (0,0), (-1,3), (1,2), and (0,5) to parallelogram D with vertices (0,0), (3,2), (1,-1), and (4,1), the key is to analyze how each vertex of D* corresponds to a vertex in D. The mapping shows that (0,0) remains unchanged, while the other points require determining the transformation matrix. By representing T as a 2x2 matrix, the entries can be calculated based on the vertex mappings. The discussion concludes with a successful identification of the transformation matrix.
toforfiltum
Messages
341
Reaction score
4

Homework Statement


If ##D^*## is the parallelogram whose vertices are ##(0,0)##,##(-1,3)##, ##(1,2)##, and ##(0,5)## and D is the parallelogram whose vertices are ##(0,0)##, ##(3,2)##,##(1,-1)## and ##(4,1)##, find a transformation ##T## such that ##T(D^*)=D##.

Homework Equations

The Attempt at a Solution


From drawing both parallelograms, the point ##(0,0)## maps to ##(0,0)##, point ##(-1,3)## maps to ##(3,2)##, point ##(0,5)## maps to ##(4,1)## and point ##(1,2)## maps to ##(1,-1)##.

I really have no idea how to figure out the transformation. I don't see any pattern at all. Any hints?

Thanks!
 
Physics news on Phys.org
toforfiltum said:

Homework Statement


If ##D^*## is the parallelogram whose vertices are ##(0,0)##,##(-1,3)##, ##(1,2)##, and ##(0,5)## and D is the parallelogram whose vertices are ##(0,0)##, ##(3,2)##,##(1,-1)## and ##(4,1)##, find a transformation ##T## such that ##T(D^*)=D##.

Homework Equations

The Attempt at a Solution


From drawing both parallelograms, the point ##(0,0)## maps to ##(0,0)##, point ##(-1,3)## maps to ##(3,2)##, point ##(0,5)## maps to ##(4,1)## and point ##(1,2)## maps to ##(1,-1)##.

I really have no idea how to figure out the transformation. I don't see any pattern at all. Any hints?

Thanks!

Represent ##T## by a ##2 \times 2## matrix, and figure out what must be the four entries of the matrix.
 
Ray Vickson said:
Represent ##T## by a ##2 \times 2## matrix, and figure out what must be the four entries of the matrix.
Ah, thanks. I've got it!
 
Question: A clock's minute hand has length 4 and its hour hand has length 3. What is the distance between the tips at the moment when it is increasing most rapidly?(Putnam Exam Question) Answer: Making assumption that both the hands moves at constant angular velocities, the answer is ## \sqrt{7} .## But don't you think this assumption is somewhat doubtful and wrong?

Similar threads

Replies
9
Views
3K
  • · Replies 0 ·
Replies
0
Views
1K
Replies
0
Views
1K
  • · Replies 7 ·
Replies
7
Views
2K
  • · Replies 3 ·
Replies
3
Views
1K
Replies
1
Views
4K
  • · Replies 7 ·
Replies
7
Views
2K
  • · Replies 3 ·
Replies
3
Views
2K
  • · Replies 1 ·
Replies
1
Views
1K
Replies
4
Views
2K